5
$\begingroup$

(In what follows, Freiling's Axiom of Symmetry is simply the following:

($A_{\aleph_0}$) :( $\forall$$f$: $\mathbf R$ $\rightarrow$$\mathbf R_{\aleph_0}$)($\exists$$x_1$,$x_2$)($x_2$$\notin$$f$($x_1$) $\land$ $x_1$$\notin$$f$($x_2$)), where $\mathbf R$ are the reals, $\mathbf R_{\aleph_0}$ is the set of all countable sets of reals, and $f$ : $\mathbf R$$\rightarrow$$\mathbf R_{\aleph_0}$ is interpreted as "$f$ assigns to each real a countable set of reals".

Note that one can replace $\mathbf R$ and $\mathbf R_{\aleph_0}$ with $[$0,1$]$ and $[$0,1$]_{\aleph_0}$.

Note also that:

$A_{f}$ = {($x_1$,$x_2$): $x_2$$\notin$$f$($x_1$)}, $A^{f}$ = {($x_1$,$x_2$): $x_1$$\notin$$f$($x_2$)}, $\forall$$f$: $[$ 0,1 $]$$\rightarrow$ $[$0,1$]_{\aleph_0}$ ($\lambda^{2}$($A_{f}$)=1 $\land$ $\lambda^{2}$($A^{f}$)=1), where $\lambda$ is the Lebesgue measure on the Borel algebra $\mathfrak B$ in $[$ 0,1 $]$, $\lambda^{2}$ is the requisite product measure; that this implies that

$\lambda^{2}$($A_{f}$ $\cap$ $A^{f}$)=1 when $\lnot$$CH$, $CH$ implies that $\lambda^{2}$($A_{f}$ $\cap$ $A^{f}$)=0, which implies that ($[$ 0,1 $]$$\times$ $[$ 0,1 $]$)$\setminus$$A_{f}$=$A^{f}$.)

A common critique of Freiling's Axiom of symmetry is "...that violations of the Axiom of Symmetry are fundamentally connected with non-measurable sets, and counterexample functions $f$ to $AS$ cannot be nice measurable functions..." (this from Prof. Hamkins' answer to Mateo Mio's mathoverflow question " Axiom of Symmetry, aka Freiling's argument against $CH$"). This is also stated by Kai Hauser in his paper "What New Axioms Could Not Be", as follows:

"...the mathematical flaw in the transition from the thought experiment to $A_{\aleph_0}$ lies in its haphazard generalization of a plausable intuition about measurable subsets of $[$ 0,1 $]$ to arbitrary subsets of $[$ 0,1 $]$."

Why does he say this? Well, in the aforementioned paper, he states that:

"The formal statement of the conclusion of Freiling's thought experiment is

$\forall$$f$ : [ 0,1 $]$ $\rightarrow$ $[$ 0,1 $]_{\aleph_0}$ ($\lambda^2$($A_{f}$)=1 $\land$ $\lambda^2$($A^{f}$)=1) ...From this it follows that $\lambda^2$($A_{f}$ $\cap$$A^{f}$)=1, hence this intersection is nonempty" . Then, in the next paragraph, he states that "this chain of inferences requires the measurability of $A_{f}$ and $A^{f}$ for which there seems to be no a priori reason...under $CH$, let $\lt$ be a well-order of the continuum in order type $\omega_1$ and define $f$(x)={y: y$\le$x}...Then, by Fubini's theorem, neither $A_{f}$ nor $A^{f}$ are measurable" [this is just the Sierpinski example--my comment].

However Freiling, in his paper, has the following lemma:

Lemma ($ZFC$+$A_{null}$). There is no set on the unit square which is null on almost every vertical line and full on almost every horizontal line, where $A_{null}$ is

$\forall$$f$:$\mathbf R$$\rightarrow$$\mathbf R_{null}$($\exists$$x_1$,$x_2$($x_1$$\notin$$f$($x_2$) $\land$ $x_2$$\notin$$f$($x_1$)), where $\mathbf R_{null}$ is the set of all sets of reals with Lebesgue measure zero.

Question: Is $ZFC$+$A_{null}$+"$\mathfrak c$ is real-valued measurable" consistent if $ZFC$+ "There exists a measurable cardinal" is consistent?

$\endgroup$
0

1 Answer 1

5
$\begingroup$

Claim: The principle $A_{null}$ follows from continuum is RVM.

Proof: Let $m$ be a total extension of Lebesgue measure. Suppose every vertical section of $A \subseteq \mathbb{R}^2$ is Lebesgue null. First check that $A$ is $m \otimes m$-null. Put $B = \{(y, x) : (x, y) \in A\}$. Then $B$ is also $m \otimes m$-null. So if $(x, y)$ is outside $A \cup B$, then $x \notin A_y$ and $y \notin A_x$.

The above argument only requires: Every countably generated sigma algebra extending the Borel algebra admits an extension of Lebesgue measure.

$\endgroup$
4
  • $\begingroup$ Why is $A$ $m \otimes m$-measurable? $\endgroup$ Sep 28, 2015 at 21:21
  • 3
    $\begingroup$ For each $e > 0$, let $U = U(e)$ be a subset of plane such that each vertical section $U_x$ is an open cover of $A_x$ of length less than $e$. Next note that each $U$ is a countable union of sets of form $A_{I, U} \times I$ where $I$ is a rational interval and $A_{I, U} = \{x : I \subseteq U_x\}$. So it suffices for $m$ to measure $A_{I, U}$ for rational $e > 0$. $\endgroup$
    – Ashutosh
    Sep 28, 2015 at 22:24
  • $\begingroup$ @MonroeEskew: Have you a counterexample to this? $\endgroup$ Sep 28, 2015 at 22:54
  • $\begingroup$ No, Ashutosh is right. $\endgroup$ Sep 28, 2015 at 23:09

Your Answer

By clicking “Post Your Answer”, you agree to our terms of service and acknowledge you have read our privacy policy.

Not the answer you're looking for? Browse other questions tagged or ask your own question.